Đến nội dung

Hình ảnh

VMF's Marathon Bất Đẳng Thức Olympic

marathon aops vmf

  • Please log in to reply
Chủ đề này có 162 trả lời

#61
Gachdptrai12

Gachdptrai12

    Thượng sĩ

  • Điều hành viên THCS
  • 280 Bài viết

Lời giải bài 21. Lời giải cuả mình phần đầu giống anh Huyện nên mình bỏ phần đầu nhé
Ta chứng minh
$\sum \frac{5}{2a^2+\sqrt{4a^4+5abc}}\ge 3$
Tuy nhiên áp dụng C-S
${\sum \frac{5}{2a^2+\sqrt{4a^4+5abc}}=\sum \frac{5b^2c^2}{2a^2b^2c^2+b^2c^2\sqrt{4a^4+5abc}}\ge \frac{5(ab+bc+ca)^2}{6a^2b^2c^2+\sum b^2c^2\sqrt{4a^4+5abc}}}$
Ta sẽ chứng minh
$5(\frac{1}{a}+\frac{1}{b}+\frac{1}{c})^2\ge 18+\sum 3\frac{\sqrt{4a^4+5abc}}{a^2}$
Áp dụng AM-GM
$3\frac{\sqrt{4a^4+5abc}}{a^2}\le \frac{9+4a^4+5abc}{2a^2}$
Ta chứng minh$\sum \frac{1}{2a^2}+10\sum \frac{1}{ab}\ge 2\sum a^2+3+\frac{5}{2}\sum \frac{1}{a}\sum ab$
Là 1 bất đẳng thức đúng khi $a+b+c=3$


Bài viết đã được chỉnh sửa nội dung bởi hoanglong2k: 31-05-2016 - 23:06


#62
quykhtn-qa1

quykhtn-qa1

    Binh nhất

  • Thành viên
  • 23 Bài viết
Lời giải của bạn Nguyenhuyen_AG và Gachdptrai12 cho bài 21 là chuẩn xác rồi với ý tưởng chính là sử dụng bất đẳng thức Schur.
 
Bài toán 22. (AoPS) Cho các số thực dương $a,b,c,d$. Chứng minh rằng
\[ a^4+b^4+c^4+d^4-4abcd\geq 2a^2|(d-b)(d-c)|.\]

Bài viết đã được chỉnh sửa nội dung bởi hoanglong2k: 31-05-2016 - 23:06


#63
hoanglong2k

hoanglong2k

    Trung úy

  • Điều hành viên THCS
  • 965 Bài viết

 

Bài toán 22. (AoPS) Cho các số thực dương $a,b,c,d$. Chứng minh rằng
\[ a^4+b^4+c^4+d^4-4abcd\geq 2a^2|(d-b)(d-c)|.\]

 

 Lời giải bài 22.

 - Trường hợp $(d-b)(d-c)\leq 0$,ta viết bất đẳng thức lại là $a^4+b^4+c^4+d^4-4abcd\geq 2a^2(d-b)(c-d)$

   Áp dụng bất đẳng thức AM-GM ta có $(d-b)(c-d)\leq \dfrac{(b-c)^2}{4}$

   Nên ta chỉ cần chứng minh $a^4+b^4+c^4+d^4-4abcd\geq \dfrac{a^2(b-c)^2}{2}$

   Tương đương $2(a^4+b^4+c^4+d^4)+2a^2bc\geq a^2b^2+a^2c^2+8abcd$

   Áp dụng bất đẳng thức AM-GM ta có $a^2bc+a^2bc+d^4+b^2c^2\geq 4abcd$ và $a^4+b^4+c^4+d^4\geq 4abcd$

   Nên ta chỉ cần chứng minh $a^4+b^4+c^4\geq a^2b^2+b^2c^2+c^2a^2$, hiển nhiên đúng

 - Trường hợp $(d-b)(d-c)\geq 0$, áp dụng bất đẳng thức AM-GM ta sẽ có $b^4+c^4\geq \dfrac{(b+c)^4}{8},bc\leq \dfrac{(b+c)^2}{4},(d-b)(d-c)\leq \dfrac{(2d-b-c)^2}{4}$

   Đặt $t=\dfrac{b+c}{2}$ thì ta chỉ cần chứng minh $a^4+2t^4+d^4\geq 2a^2(d-t)^2$

   Tương đương với $(a^2-d^2)^2+2t(t-a)(t^2+at-2ad)\geq 0$

   Hiển nhiên với $t=\max \{a,t,d\}$ hoặc $t=\min \{a,t,d\}$  thì bất đẳng thức trên đúng nên ta chỉ xét khi $t$ nằm giữa $a$ và $d$

   Nếu $a\geq t\geq d$, đặt $a=d+x$ và $t=d+y$ thì ta cần chứng minh $(x^2+2dx)^2+2(d+y)(y-x)(3dy+y^2+xy-dx)\geq 0$

   Tương đương với $2d^2(3x^2-4xy+3y^2)+4d(x^3-2xy^2+2y^3)+x^4-2x^2y^2+2y^4\geq 0$

   Hiển nhiên đúng do $3x^2-4xy+3y^2\geq x^2+y^2\geq 0,x^3-2xy^2+2y^3\geq xy^2\geq 0,x^4-2x^2y^2+2y^4\geq y^4\geq 0$

   Nếu $a\leq t\leq d$, đặt $d=a+u,t=a+v$ thì ta cần chứng minh $a^2(4u^2-4uv+6v^2)+4a(u^3-uv^2+2v^3)+u^4+2v^4\geq 0$, đúng

 Vậy ta có điều cần chứng minh. Dấu "=" xảy ra khi $a=b=c=d$

 

 Bài toán 23. (Sưu tầm) Cho các số thực không âm $x,y,z$ thỏa mãn $x^2+y^2+z^2+2xyz=1$. Chứng minh rằng

\[x^3y+y^3z+z^3x\leq \dfrac{3\sqrt{3}}{16}\]


Bài viết đã được chỉnh sửa nội dung bởi hoanglong2k: 01-06-2016 - 13:09


#64
quykhtn-qa1

quykhtn-qa1

    Binh nhất

  • Thành viên
  • 23 Bài viết

 

Bài toán 22. (AoPS) Cho các số thực dương $a,b,c,d$. Chứng minh rằng
\[ a^4+b^4+c^4+d^4-4abcd\geq 2a^2|(d-b)(d-c)|.\]

 

Lời giải của em cho bài 22 cũng khá tốt rồi mặc dù còn nhiều tính toán ở trường hợp 2, hoanglong2k. Lời giải dưới đây tính toán đơn giản hơn:
 
Lời giải bài 22. Xét $2$ trường hợp:
Trường hợp 1. Nếu $(d-b)(d-c)\leq 0$, ta có 
$$|(d-b)(d-c)|=(b-d)(d-c)\leq \dfrac{(b-d+d-c)^2}{4}=\dfrac{(b-c)^2}{4}.$$
Ta chỉ cần chứng minh 
\[ a^4+b^4+c^4+d^4-4abcd \geq \dfrac{a^2(b-c)^2}{2},\]
\[ a^4+b^4+c^4+d^4+2a^2bc\geq \dfrac{a^2(b+c)^2}{2}+4abcd.\]
Sử dụng bất đẳng thức AM-GM, ta có
\[ a^2bc+a^2bc+b^2c^2+d^4\geq 4abcd,\]
\[ a^4+b^4+c^4\geq a^2b^2+b^2c^2+c^2a^2,\]
\[a^2(b^2+c^2)\geq \dfrac{a^2(b+c)^2}{2}.\]
Cộng theo vế các bất đẳng thức trên ta có điều phải chứng minh.
 
Trường hợp 2. Nếu $(d-b)(d-c)>0$. Do tính thuần nhất, ta có thể chuẩn hóa $b+c=2$. Sử dụng bất đẳng thức AM-GM, ta có
$$|(d-b)(d-c)|=(d-b)(d-c)\leq \left(d-\dfrac{b+c}{2}\right)^2=(d-1)^2,$$
$$b^4+c^4\geq 4b-3+4c-3=2,$$
$$ abcd\leq \dfrac{a^2+1}{2}\cdot\dfrac{(b+c)^2}{4}\cdot d=\dfrac{(a^2+1)d}{2}.$$
Do đó, ta chỉ cần chứng minh
\[a^4+2+d^4-2(a^2+1)d \geq 2a^2(d-1)^2,\]
\[a^4-2a^2(d^2-d+1)+d^4-2d+2,\]
\[(a^2+d-d^2-1)^2+(2d+1)(d-1)^2\geq 0.\]
Bất đẳng thức trên hiển nhiên đúng. Phép chứng minh hoàn tất. Đẳng thức xảy ra khi và chỉ khi $a=b=c=d$.

Bài viết đã được chỉnh sửa nội dung bởi quykhtn-qa1: 02-06-2016 - 21:54


#65
quykhtn-qa1

quykhtn-qa1

    Binh nhất

  • Thành viên
  • 23 Bài viết

Bài toán 23. (Sưu tầm) Cho các số thực không âm $x,y,z$ thỏa mãn $x^2+y^2+z^2+2xyz=1$. Chứng minh rằng

\[x^3y+y^3z+z^3x\leq \dfrac{3\sqrt{3}}{16}\]

Về cơ bản thì bài toán này có thể dử dụng ý tưởng như bài  toán 19, 20. Tất nhiên bài này lời giải khá đơn giản vì chỉ có 1 dấu đẳng thức xảy ra (không tính hoán vị).



#66
Gachdptrai12

Gachdptrai12

    Thượng sĩ

  • Điều hành viên THCS
  • 280 Bài viết

 Bài toán 23. (Sưu tầm) Cho các số thực không âm $x,y,z$ thỏa mãn $x^2+y^2+z^2+2xyz=1$. Chứng minh rằng

\[x^3y+y^3z+z^3x\leq \dfrac{3\sqrt{3}}{16}\]

ta chỉ cần chứng minh bdt khi $x\geq y\geq z$ 

ta có $x\geq \frac{1}{2}\Rightarrow 1=x^{2}+y^{2}+z^{2}+2xyz\geq x^{2}+y^{2}+z^{2}+yz$

từ đó ta có $x^{3}y+y^{3}z+z^{3}x\leq xy(x^{2}+xy+y^{2})=S$

mà theo bddt AM-GM ta có

$S^{2}=x^{2}y^{2}(x^{2}+zy+z^{2})^{2}=27\frac{x^{2}}{3}y^{2}(\frac{x^{2}+yz+z^{2}}{3})^{2}\leq \frac{27}{256}(\frac{x^{2}}{3}+\frac{2(x^{2}+yz+z^{2})}{3}+y^{2})^{4}= \frac{27}{256}(x^{2}+y^{2}+\frac{2(yz+z^{2})}{3})^{4}\leq \frac{27}{256}(x^{2}+y^{2}+z^{2}+yz)^{4}\leq \frac{27}{256}\Rightarrow S\leq \frac{3\sqrt{3}}{16}$

vậy ta có đpcm

 

Về cơ bản thì bài toán này có thể dử dụng ý tưởng như bài  toán 19, 20. Tất nhiên bài này lời giải khá đơn giản vì chỉ có 1 dấu đẳng thức xảy ra (không tính hoán vị).

em nghĩ nó không dễ đâu ạ dù chỉ có 1 dấu bằng nhưng rất khó đoán phải dùng máy tính :3 :v

em đang suy nghĩ 1 cách khác đó là chứng minh $f(\sqrt{\frac{1-z}{2}},\sqrt{\frac{1-z}{2}},z)\geq \frac{3\sqrt{3}}{16}$


Bài viết đã được chỉnh sửa nội dung bởi Gachdptrai12: 04-06-2016 - 21:03


#67
quykhtn-qa1

quykhtn-qa1

    Binh nhất

  • Thành viên
  • 23 Bài viết

ta chỉ cần chứng minh bdt khi $x\geq y\geq z$ 

ta có $x\geq \frac{1}{2}\Rightarrow 1=x^{2}+y^{2}+z^{2}+2xyz\geq x^{2}+y^{2}+z^{2}+yz$

từ đó ta có $x^{3}y+y^{3}z+z^{3}x\leq xy(x^{2}+xy+y^{2})=S$

mà theo bddt AM-GM ta có

$S^{2}=x^{2}y^{2}(x^{2}+zy+z^{2})^{2}=27\frac{x^{2}}{3}(\frac{x^{2}+yz+z^{2}}{3})^{2}\leq = \frac{27}{256}(\frac{x^{2}}{3}+\frac{2(x^{2}+yz+z^{2})}{3}+y^{2})^{4}= \frac{27}{256}(x^{2}+y^{2}+\frac{2(yz+z^{2})}{3})^{4}\leq \frac{27}{256}(x^{2}+y^{2}+z^{2}+yz)^{4}\leq \frac{27}{256}\Rightarrow S\leq \frac{3\sqrt{3}}{16}$

vậy ta có đpcm

 

em nghĩ nó không dễ đâu ạ dù chỉ có 1 dấu bằng nhưng rất khó đoán phải dùng máy tính :3 :v

em đang suy nghĩ 1 cách khác đó là chứng minh $f(\sqrt{\frac{1-z}{2}},\sqrt{\frac{1-z}{2}},z)\geq \frac{3\sqrt{3}}{16}$

 

Lời giải này ý tưởng đúng nhưng gõ sai nhiều chỗ, em nên viết nghiêm túc hơn. Ý tưởng lời giải anh nói ở trên là quy về chứng minh bất đẳng thức $1$ biến:

$$ x^3y+y^3z+z^3x \leq y(x^3+z^3+xyz) \leq y(x^2+z^2+2xyz)^{\frac{3}{2}}=y(1-y^2)^{\frac{3}{2}} \leq \frac{3\sqrt{3}}{16}.$$
Phần chứng minh $x^3+z^3+xyz\leq (x^2+z^2+2xyz)^{\frac{3}{2}}$ cơ bản là giống với lời giải của em. Chú ý đây là bất đẳng thức hoán vị nên không dùng dồn biến kiểu quy về chứng minh $f(\sqrt{\frac{1-z}{2}},\sqrt{\frac{1-z}{2}},z)\leq \frac{3\sqrt{3}}{16}$ như em nói được. 


Bài viết đã được chỉnh sửa nội dung bởi quykhtn-qa1: 03-06-2016 - 21:30


#68
Gachdptrai12

Gachdptrai12

    Thượng sĩ

  • Điều hành viên THCS
  • 280 Bài viết

Lời giải này ý tưởng đúng nhưng gõ sai nhiều chỗ, em nên viết nghiêm túc hơn. Ý tưởng lời giải anh nói ở trên là quy về chứng minh bất đẳng thức $1$ biến:

$$ x^3y+y^3z+z^3x \leq y(x^3+z^3+xyz) \leq y(x^2+z^2+2xyz)^{\frac{3}{2}}=y(1-y^2)^{\frac{3}{2}} \leq \frac{3\sqrt{3}}{16}.$$
Phần chứng minh $x^3+z^3+xyz\leq (x^2+z^2+2xyz)^{\frac{3}{2}}$ cơ bản là giống với lời giải của em. Chú ý đây là bất đẳng thức hoán vị nên không dùng dồn biến kiểu quy về chứng minh $f(\sqrt{\frac{1-z}{2}},\sqrt{\frac{1-z}{2}},z)\leq \frac{3\sqrt{3}}{16}$ như em nói được. 

 

chắc có lẽ cách em sai 



#69
ineX

ineX

    Sĩ quan

  • Thành viên
  • 353 Bài viết

Vì không thấy ai đăng tiếp, mình xin góp một bài, xin lấy tiêu đề là bài toán 24, nếu các bạn không đồng ý thì có thể bỏ bài toán này!

 

Bài toán 24: (Nguồn: Juliel)

Cho các số thực dương a,b,c. Chứng minh:

aaa.PNG

 


Bài viết đã được chỉnh sửa nội dung bởi ineX: 04-06-2016 - 23:09

"Tôi sinh ra là để thay đổi thế giới chứ không phải để thế giới thay đổi tôi" - Juliel

 

3cf67218ea144a6eb6caf571068071ff.1.gif


#70
Gachdptrai12

Gachdptrai12

    Thượng sĩ

  • Điều hành viên THCS
  • 280 Bài viết

Vì không thấy ai đăng tiếp, mình xin góp một bài, xin lấy tiêu đề là bài toán 24, nếu các bạn không đồng ý thì có thể bỏ bài toán này!

 

Bài toán 24: (Nguồn: Juliel)

Cho các số thực dương a,b,c. Chứng minh:

$$\sqrt[7]{\frac{a^{7}+b^{7}}{2}}+\sqrt[7]{\frac{b^{7}+c^{7}}{2}}+\sqrt[7]{\frac{c^{7}+a^{7}}{2}}\leq \left ( a+b+c \right )^{10}\left ( \frac{1}{9a}+\frac{1}{9b}+\frac{1}{9c} \right )^{9}$$

 

Áp dụng BĐT AM-GM :

$\dfrac{a^7+b^7}{(a+b)^6}+\dfrac{a+b}{2^6}+\dfrac{a+b}{2^6}+\dfrac{a+b}{2^6}+\dfrac{a+b}{2^6}+\dfrac{a+b}{2^6}+\dfrac{a+b}{2^6}\geq 7\sqrt[7]{\dfrac{a^7+b^7}{(2^6)^6}}=\dfrac{7}{32}\sqrt[7]{\dfrac{a^7+b^7}{2}}$

Tức là :
$\dfrac{a^7+b^7}{(a+b)^6}+\dfrac{3}{32}(a+b)\geq \dfrac{7}{32}\sqrt[7]{\dfrac{a^7+b^7}{2}}$
Ta có :
$(a+b)^7=a^7+b^7+7ab(a^5+b^5)+21a^2b^2(a^3+b^3)+35a^3b^3(a+b)=a^7+b^7+7ab\left [ a^5+b^5+3ab(a^3+b^3)+5a^2b^2(a+b) \right ]$
Theo AM-GM :
$12a^5+9b^5+7a^4b+2ab^4\geq 30\sqrt[30]{(a^5)^{12}.(b^5)^9.(a^4b)^7.(ab^4)^2}=30a^3b$
$12b^5+9a^5+7b^4a+2ba^4\geq 30\sqrt[30]{(b^5)^{12}.(a^5)^9.(b^4a)^7.(ba^4)^2}=30b^3a$
Cộng vế theo vế hai kết quả trên thì được :
$7(a^5+b^5)+3ab(a^3+b^3)\geq 10a^2b^2(a+b)$
Suy ra :
 
$16(a^5+b^5)+48ab(a^3+b^3)+80a^2b^2(a+b)\geq 9(a^5+b^5)+45ab(a^3+b^3)+90a^2b^2(a+b)\Leftrightarrow 16\left [ a^5+b^5+3ab(a^3+b^3)+5a^2b^2(a+b) \right ]\geq 9\left [ a^5+b^5+5ab(a^3+b^3)+10a^2b^2(a+b) \right ]\Leftrightarrow a^5+b^5+3ab(a^3+b^3)+5a^2b^2(a+b) \geq \dfrac{9}{16}(a+b)^5$
Từ đó ta được :
 
$(a+b)^7\geq a^7+b^7+7ab.\dfrac{9}{16}(a+b)^5\Leftrightarrow \dfrac{a^7+b^7}{(a+b)^6}\leq a+b-\dfrac{63ab}{16(a+b)}$
Lại được tiếp :
$\dfrac{7}{32}\sqrt[7]{\dfrac{a^7+b^7}{2}}\leq \dfrac{3}{32}(a+b)+(a+b)-\dfrac{63ab}{16(a+b)}=\dfrac{35}{32}(a+b)-\dfrac{63ab}{16(a+b)}\Leftrightarrow \sqrt[7]{\dfrac{a^7+b^7}{2}}\leq 5(a+b)-\dfrac{18ab}{a+b}$
Thiết lập các BĐT tương tự rồi cộng chúng vế theo vế, ta dẫn đến việc chứng minh :
$\left ( a+b+c \right )^{10}\left ( \dfrac{1}{9a}+\dfrac{1}{9b} +\dfrac{1}{9c}\right )^9+\dfrac{18ab}{a+b}+\dfrac{18bc}{b+c}+\dfrac{18ca}{c+a}\geq 10(a+b+c)\Leftrightarrow \dfrac{(a+b+c)^{10}.(ab+bc+ca)^9}{(abc)^9.9^9}+\dfrac{18}{\dfrac{1}{a}+\dfrac{1}{b}}+\dfrac{18}{\dfrac{1}{b}+\dfrac{1}{c}}+\dfrac{18}{\dfrac{1}{c}+\dfrac{1}{a}}\geq 10(a+b+c)$
Theo Cauchy-Schwarz :
$\dfrac{18}{\dfrac{1}{a}+\dfrac{1}{b}}+\dfrac{18}{\dfrac{1}{b}+\dfrac{1}{c}}+\dfrac{18}{\dfrac{1}{c}+\dfrac{1}{a}}\geq \dfrac{18.9}{2\left ( \dfrac{1}{a}+\dfrac{1}{b} +\dfrac{1}{c}\right )}=\dfrac{81abc}{ab+bc+ca}$
Như vậy cần chỉ ra :
$\dfrac{(a+b+c)^{10}.(ab+bc+ca)^9}{(abc)^9.9^9}+\dfrac{81abc}{ab+bc+ca}\geq 10(a+b+c)$
Nhưng điều này hiển nhiên đúng theo AM-GM cho 10 số :
$\dfrac{(a+b+c)^{10}.(ab+bc+ca)^9}{(abc)^9.9^9}+\dfrac{81abc}{ab+bc+ca}=\dfrac{(a+b+c)^{10}.(ab+bc+ca)^9}{(abc)^9.9^9}+9.\dfrac{9abc}{ab+bc+ca}\geq 10\sqrt[10]{\dfrac{(a+b+c)^{10}.(ab+bc+ca)^9}{(abc)^9.9^9}.\dfrac{(9abc)^9}{(ab+bc+ca)^9}}=10(a+b+c)$
p/s:đề nghị sửa đề  :D  :D

Bài viết đã được chỉnh sửa nội dung bởi Gachdptrai12: 04-06-2016 - 21:18


#71
Gachdptrai12

Gachdptrai12

    Thượng sĩ

  • Điều hành viên THCS
  • 280 Bài viết

Bài toán 25. (Võ Quốc Bá Cẩn) Cho $a,b,c$ là các số thực không âm, chứng minh

$\dfrac{4(a^{2}+b^{2}+c^{2})}{ab+bc+ca}+\sqrt{3}\left(\frac{a+2b}{\sqrt{a^{2}+2b^{2}}}+\frac{b+2c}{\sqrt{b^{2}+2c^{2}}}+\frac{c+2a}{\sqrt{c^{2}+2a^{2}}}\right)\geq 13$


Bài viết đã được chỉnh sửa nội dung bởi hoanglong2k: 05-06-2016 - 21:52


#72
Nguyenngoctu

Nguyenngoctu

    Trung sĩ

  • Thành viên
  • 118 Bài viết

Bài toán 6. 

Cho a,b,c là các số thực dương thỏa mãn ab+bc+ca=3. Chứng minh rằng:

$$4\left( {{a^4} + {b^4} + {c^4}} \right) + 11abc\left( {a + b + c} \right) \ge 45$$.

Lời giải.

Ta có $$\begin{array}{l} 4\left( {{a^4} + {b^4} + {c^4}} \right) + 11abc\left( {a + b + c} \right) \ge 45\\ \Leftrightarrow 4\left( {{a^4} + {b^4} + {c^4}} \right) + 11abc\left( {a + b + c} \right) \ge 5{\left( {ab + bc + ca} \right)^2}\\ \Leftrightarrow 4\left( {{a^4} + {b^4} + {c^4}} \right) + abc\left( {a + b + c} \right) \ge 5\left( {{a^2}{b^2} + {b^2}{c^2} + {c^2}{a^2}} \right) \end{array}$$

Ta có: $$\begin{array}{l} 4\left( {{a^4} + {b^4} + {c^4}} \right) + abc\left( {a + b + c} \right) = \left[ {{a^4} + {b^4} + {c^4} + abc\left( {a + b + c} \right)} \right] + 3\left( {{a^4} + {b^4} + {c^4}} \right)\\ {a^4} + {b^4} + {c^4} + abc\left( {a + b + c} \right) \ge ab\left( {{a^2} + {b^2}} \right) + bc\left( {{b^2} + {c^2}} \right) + ca\left( {{c^2} + {a^2}} \right)\\ \ge ab.2ab + bc.2bc + ca.2ca = 2\left( {{a^2}{b^2} + {b^2}{c^2} + {c^2}{a^2}} \right) \end{array}$$.

và $$3\left( {{a^4} + {b^4} + {c^4}} \right) \ge 3\left( {{a^2}{b^2} + {b^2}{c^2} + {c^2}{a^2}} \right)$$.

Do đó $$4\left( {{a^4} + {b^4} + {c^4}} \right) + abc\left( {a + b + c} \right) \ge 5\left( {{a^2}{b^2} + {b^2}{c^2} + {c^2}{a^2}} \right)$$. Xong!



#73
fatcat12345

fatcat12345

    Binh nhất

  • Banned
  • 46 Bài viết

Lời giải bài 25.

Giả sử $a=max(a,b,c),$

$\frac{4(a^2+b^2+c^2)}{ab+bc+ca}+\sqrt{3}\sum_{cyc}\frac{a+2b}{\sqrt{a^2+2b^2}}-13$
$=\sum_{cyc}\left ( \frac{1}{ab+bc+ca}-\frac{3}{(b+2c+\sqrt{3(b^2+2c^2)})\sqrt{3(b^2+2c^2)}} \right )(b-c)^2$
$\geq [ \frac{1}{ab+bc+ca}-\frac{3}{(c+2a+\sqrt{3(c^2+2a^2)})\sqrt{3(c^2+2a^2)}} ](c-a)^2$$+[ \frac{1}{ab+bc+ca}-\frac{3}{(b+2c+\sqrt{3(b^2+2c^2)})\sqrt{3(b^2+2c^2)}}](b-c)^2$
$\geq \frac{1}{b^2}[ \frac{a^2}{ab+bc+ca}-\frac{3a^2}{(c+2a+\sqrt{3(c^2+2a^2)})\sqrt{3(c^2+2a^2)}}$$+\frac{b^2}{ab+bc+ca}-\frac{3b^2}{(b+2c+\sqrt{3(b^2+2c^2)})\sqrt{3(b^2+2c^2)}}](b-c)^2$
$\geq \frac{1}{b^2}\left ( \frac{a^2+b^2}{ab+bc+ca}-\frac{3a^2}{2(c+2a)^2}-\frac{3b^2}{2(b+2c)^2} \right )(b-c)^2$
$\geq \frac{(8a^4b^2+8a^2b^4-15a^3b^3)+(19a^4bc-19a^3b^2c)+(4a^4bc-4a^2bc^3)}{2b^2(ab+bc+ca)(c+2a)^2(b+2c)^2}(b-c)^2\geq 0\blacksquare$
 
Bài toán 26. Cho $a,b,c > 0$. Chứng minh
\[(a+b+c)\left ( \frac{1}{a}+\frac{1}{b}+\frac{1}{c} \right )\geq 1+2\sqrt[3]{6(a^2+b^2+c^2)\left ( \frac{1}{a^2}+\frac{1}{b^2}+\frac{1}{c^2} \right )+10}.\]
 

Bài viết đã được chỉnh sửa nội dung bởi hoanglong2k: 05-06-2016 - 21:54


#74
Gachdptrai12

Gachdptrai12

    Thượng sĩ

  • Điều hành viên THCS
  • 280 Bài viết

Bài 25.

Giả sử $a\geq b\geq c,$

$\frac{4(a^2+b^2+c^2)}{ab+bc+ca}+\sqrt{3}\sum_{cyc}\frac{a+2b}{\sqrt{a^2+2b^2}}-13$
$=\sum_{cyc}\left ( \frac{1}{ab+bc+ca}-\frac{3}{(b+2c+\sqrt{3(b^2+2c^2)})\sqrt{3(b^2+2c^2)}} \right )(b-c)^2$
$\geq \left ( \frac{1}{ab+bc+ca}-\frac{3}{(c+2a+\sqrt{3(c^2+2a^2)})\sqrt{3(c^2+2a^2)}} \right )(c-a)^2+\left ( \frac{1}{ab+bc+ca}-\frac{3}{(b+2c+\sqrt{3(b^2+2c^2)})\sqrt{3(b^2+2c^2)}} \right )(b-c)^2$
$\geq \frac{1}{b^2}\left ( \frac{a^2}{ab+bc+ca}-\frac{3a^2}{(c+2a+\sqrt{3(c^2+2a^2)})\sqrt{3(c^2+2a^2)}}+\frac{b^2}{ab+bc+ca}-\frac{3b^2}{(b+2c+\sqrt{3(b^2+2c^2)})\sqrt{3(b^2+2c^2)}} \right )(b-c)^2$
$\geq \frac{1}{b^2}\left ( \frac{a^2+b^2}{ab+bc+ca}-\frac{3a^2}{2(c+2a)^2}-\frac{3b^2}{2(b+2c)^2} \right )(b-c)^2$
$\geq \frac{(8a^4b^2+8a^2b^4-15a^3b^3)+(19a^4c^2-19a^3b^2c)+(4a^4bc-4a^2bc^3)}{2b^2(ab+bc+ca)(c+2a)^2(b+2c)^2}\geq 0\blacksquare$

 Bách coi lại bài này đâu thể giả sử $a\geq b\geq c$ và cái tô đỏ thứ 2 là chưa chắc lớn hơn



#75
fatcat12345

fatcat12345

    Binh nhất

  • Banned
  • 46 Bài viết

 Bách coi lại bài này đâu thể giả sử $a\geq b\geq c$ và cái tô đỏ thứ 2 là chưa chắc lớn hơn

Đã sửa  :D !



#76
Ngockhanh99k48

Ngockhanh99k48

    Trung sĩ

  • Thành viên
  • 127 Bài viết

Lời giải bài 26: 

Đặt $(a+b+c)(\frac{1}{a} + \frac{1}{b} + \frac{1}{c}) = t$, theo AM-GM ta suy ra $t \geq 9$. 

Theo Cauchy-Schwarz ta có:

$(a+b+c)(\frac{1}{a}+\frac{1}{b}+\frac{1}{c}) = \sqrt{[(a^2+b^2+c^2)+2(ab+bc+ca)][(\frac{1}{a^2}+\frac{1}{b^2}+\frac{1}{c^2})+2(\frac{1}{ab}+\frac{1}{bc}+\frac{1}{ca})]} \geq \sqrt{(a^2+b^2+c^2)(\frac{1}{a^2}+\frac{1}{b^2}+\frac{1}{c^2})} + 2\sqrt{(ab+bc+ca)(\frac{1}{ab}+\frac{1}{bc}+\frac{1}{ca})} = \sqrt{(a^2+b^2+c^2)(\frac{1}{a^2}+\frac{1}{b^2}+\frac{1}{c^2})} + 2\sqrt{(a+b+c)(\frac{1}{a}+\frac{1}{b}+\frac{1}{c})}$. 

Do đó $(a^2+b^2+c^2)(\frac{1}{a^2}+\frac{1}{b^2}+\frac{1}{c^2}) \leq (t-2\sqrt{t})^2 = t(\sqrt{t}-2)^2$.

Điều đó có nghĩa ta chỉ cần chứng minh: 

$t \geq 1+2\sqrt[3]{6t(\sqrt{t}-2)^2+10}$ $\Leftrightarrow$ $t^3-51t^2+192t\sqrt{t}-189t-81 \geq 0$ $\Leftrightarrow$ $(\sqrt{t}-3)^3(t\sqrt{t}+9t+3\sqrt{t}+3) \geq 0$ (luôn đúng với mọi $t \geq 9$). Ta có đpcm $\blacksquare$.

P/s: bạn nào post bài mới hộ mình đi, mình chưa có bài đề xuất nào.


Bài viết đã được chỉnh sửa nội dung bởi Ngockhanh99k48: 05-06-2016 - 15:48


#77
hoanglong2k

hoanglong2k

    Trung úy

  • Điều hành viên THCS
  • 965 Bài viết

Nguyenngoctu

 

fatcat12345

  Tiện thể mình sẽ đề xuất bài toán 27 để chúng ta tiếp tục Topic :)

 Bài toán 27. (Sưu tầm) Chứng minh rằng với các số thực $x\geq 0$ tùy ý và số nguyên dương $n$ ta luôn có bất đẳng thức \[\left[nx\right]\geq \dfrac{\left[x\right]}{1}+\dfrac{\left[2x\right]}{2}+\dfrac{\left[3x\right]}{3}+\cdots +\dfrac{\left[nx\right]}{n}\]



#78
quykhtn-qa1

quykhtn-qa1

    Binh nhất

  • Thành viên
  • 23 Bài viết

Bài toán 25. (Võ Quốc Bá Cẩn) Cho $a,b,c$ là các số thực không âm, chứng minh

$\dfrac{4(a^{2}+b^{2}+c^{2})}{ab+bc+ca}+\sqrt{3}\left(\frac{a+2b}{\sqrt{a^{2}+2b^{2}}}+\frac{b+2c}{\sqrt{b^{2}+2c^{2}}}+\frac{c+2a}{\sqrt{c^{2}+2a^{2}}}\right)\geq 13$

Lời giải bài $25$ của bạn fatcat12345 chưa chính xác. Chú ý với $a=\max\{a,b,c\}$, bất đẳng thức $(a-c)^2 \geq \dfrac{a^2(b-c)^2}{b^2}$ không chính xác (chẳng hạn cho $a=c=2,\ b=1)$. Bài toán $25$ có thể chứng minh đơn giản bằng Cauchy-Schwarz và AM-GM như sau: 
\[\begin{aligned} \sum \dfrac{a+2b}{\sqrt{a^2+2b^2}} & \geq \dfrac{\left[\sum (a+2b)\right]^2}{\sum\big[ (a+2b)\sqrt{a^2+2b^2}\big]}=\dfrac{9(a+b+c)^2}{\sum \big[ (a+2b)\sqrt{a^2+2b^2}\big]} \\ & \geq \dfrac{9(a+b+c)^2}{\sum \dfrac{3(a^2+2b^2)+(a+2b)^2}{2\sqrt{3}}}=\dfrac{9\sqrt{3}(a+b+c)^2}{7(a^2+b^2+c^2)+2(ab+bc+ca)}.\end{aligned}\]
Ta chỉ cần chứng minh
$$\dfrac{4(a^2+b^2+c^2)}{ab+bc+ca}+\dfrac{27(a+b+c)^2}{7(a^2+b^2+c^2)+2(ab+bc+ca)}\geq 13,$$
$$\dfrac{28(a^2+b^2+c^2-ab-bc-ca)^2}{(ab+bc+ca)\big[7(a^2+b^2+c^2)+2(ab+bc+ca)\big]}\geq 0.$$
Bất đẳng thức trên hiển nhiên đúng, ta có điều phải chứng minh. Đẳng thức xảy ra khi và chỉ khi $a=b=c$.
 
P/S. Lời giải của bạn Ngockhanh99k48 cho bài 26 chính xác rồi.
 


#79
Nguyenhuyen_AG

Nguyenhuyen_AG

    Trung úy

  • Thành viên nổi bật 2016
  • 945 Bài viết
Bài toán 26. Cho $a,b,c > 0$. Chứng minh
\[(a+b+c)\left ( \frac{1}{a}+\frac{1}{b}+\frac{1}{c} \right )\geq 1+2\sqrt[3]{6(a^2+b^2+c^2)\left ( \frac{1}{a^2}+\frac{1}{b^2}+\frac{1}{c^2} \right )+10}.\]

 

Đặt $x = \frac{a}{b}+\frac{b}{c}+\frac{c}{a},\;y=\frac{b}{a}+\frac{c}{b}+\frac{a}{c}$ bất đẳng thức cần chứng minh trở thành

\[x+y+2 \geqslant 2\sqrt[3]{3(x+y-2)^2+3(x-y)^2+16},\]

hay là

\[(x+y+2)^3-24(x+y-2)^2-128 \geqslant 24(x-y)^2,\]

hoặc

\[(x+y-6)^3 \geqslant 24(x-y)^2.\]

Chú ý rằng

\[x+y-6 = \sum \frac{(a-b)^2}{ab} \geqslant 3\sqrt[3]{\frac{(a-b)^2(b-c)^2(c-a)^2}{a^2b^2c^2}} = 3\sqrt[3]{(x-y)^2}.\]

Từ đó suy ra điều phải chứng minh.


Bài viết đã được chỉnh sửa nội dung bởi Nguyenhuyen_AG: 06-06-2016 - 20:16

Nguyen Van Huyen
Ho Chi Minh City University Of Transport

#80
lenhatsinh3

lenhatsinh3

    Hạ sĩ

  • Thành viên
  • 86 Bài viết

Một lời giải bài toán 17: Thực chất bài tổng quát của bài toán 17 bài toán sau

$x^{n}y+y^{n}z+z^{n}x\leq \frac{n^{n}}{(n+1)^{n+1}}(x+y+z)^{n+1}$$\forall x,y,z\geq 0$,$n\geq 2$

Ta chỉ cần chứng minh bài toán tổng quát .

 Không mất tính tổng quát, giả sử $x=\max \left \{ x,y,z \right \}$ . Ta có

$\left\{\begin{matrix} y\leq x\Rightarrow y^{n}z\leq x^{n-1}yz & & \\ z\leq x\Rightarrow z^{n}x\leq zx^{n} & & \\ z^{n}x\leq z^{2}x^{n-1}& & \\ n> 1\Rightarrow \frac{n-1}{n}\geq \frac{1}{2}\Rightarrow \frac{n-1}{n}z\geq \frac{z}{2}& & \end{matrix}\right.$

Đặt $P=x^{n}y+y^{n}z+z^{n}x\leq x^{n}y+x^{n-1}yz+\frac{1}{2}z^{n}x+\frac{1}{2}z^{n}x$

    $\leq x^{n}y+x^{n-1}yz+\frac{1}{2}x^{n}z+\frac{1}{2}z^{2}x^{n-1}$

    $=x^{n-1}(x+z)\left ( y+\frac{z}{2} \right )$

    $\leq x^{n-1}(x+z)(y+\frac{n-1}{n}z)=n^{n}[\frac{x}{n}.\frac{x}{n}...\frac{x}{n}.\frac{x+z}{n}(y+\frac{n-1}{n}z)]$

    $\leq$$n^{n}[\frac{(n-1).\frac{x}{n}+\frac{x+z}{n}+y+\frac{n-1}{n}z}{n=1}]^{n+1}$

    $=\frac{n^{n}}{(n+1)^{n+1}}(x+y+z)^{n+1}$

Đẳng thức xảy ra khi và chỉ khi $\left ( x,y,z \right )\sim\left ( n,1,0 \right )$ các hoán vị của nó

Áp dụng ta có được $x^{3}y+y^{3}z+z^{3}x\leq \frac{27}{256}(x+y+z)^{4}$


Bài viết đã được chỉnh sửa nội dung bởi lenhatsinh3: 07-06-2016 - 07:44

:ukliam2:  :ukliam2:  :ukliam2:  :ukliam2:  :ukliam2:

      :ukliam2:

            :ukliam2:

                  :ukliam2:

             :ukliam2:

        :ukliam2:  

     :ukliam2:  :ukliam2:  :ukliam2:  :ukliam2:  :ukliam2:






Được gắn nhãn với một hoặc nhiều trong số những từ khóa sau: marathon, aops, vmf

0 người đang xem chủ đề

0 thành viên, 0 khách, 0 thành viên ẩn danh